Can someone help with 13 and 14

Can Someone Help With 13 And 14

Answers

Answer 1

Step-by-step explanation:

Question 13 :-

-4c³ . 7d² . 2c -²( -4c ³ - ² ) . 7d² -4c . 7d²-28cd²

Question 14 :-

n-⁴ w⁰n-⁴ . 1 n -⁴
Answer 2

Answer:

13.) -56cd²

14.) n-⁴

Step-by-step explanation:

Question 13 :- -4c³• 7d² • 2c-²

-4c³• 7d² • 2c-²

Calculate the products.

-4•7•2 c³ d²c-²-56c³ d²c-²

Multiplying the terms with the same base by adding their exponents.

-56c³-²d²

Calculate the exponents.

-56cd²

Question 14 :- n-⁴w⁰

n-⁴w⁰

Any non - zero expression raised to the power 0 equals 1.

n-⁴ 1

Any expression multiplied by 1 remains the same.

n-⁴

Related Questions

A manufacturer knows that their items have a normally distributed length, with a mean of 10.2 inches, and standard deviation of 1.8 inches. If 25 items are chosen at random, what is the probability that their mean length is less than 9.8 inche

Answers

Answer:

P [ X ≤  9.8 ]  = 0.1335

Step-by-step explanation:

P [ X ≤  9.8 ]  =  [ ( 9.8  - 10.2 )/1.8√25 ]

P [ X ≤  9.8 ]  =  - 0.4*5/1.8

P [ X ≤  9.8 ]  = - 2 / 1.8

P [ X ≤  9.8 ]  =  - 1.11

From z- table we get:  α  =  0.1335

P [ X ≤  9.8 ]  = 0.1335   or   P [ X ≤  9.8 ]  = 0.1335

What is the order of rotational symmetry of a regular pentagon?

Answers

Answer:

yeet

Step-by-step explanation:

urmom

Sorry just needed points

which of the following equations describes the line
check all that apply (1,9) (-2, 3) A. y - 1 = 2(X-9) B. Y- 2 = 2(x-3) c. y - 3 = 2(x + 2) D. Y-3 = 2(x - 2) E. y + 2 = 2(x-3) F. - 9 = 2(x - 1)​

Answers

The equation of the line that connects the two points (1,9) and (-2,3) is,

y = 2x+7

or, y-3=2x+4

or, y-3=2(x+2)

So, option c, is the correct option

Answered by GAUTHMATH

Paul rolls a fair dice 132 times.
How many times would Paul expect to roll a three?

Answers

There are 6 numbers on the dice, rolling a 3 would be 1/6

Multiply total rolls by probability of getting a 3:

132 x 1/6 = 132/6 = 22

The answer: 22 times.

Un par de zapatos más dos pantalones valen $ 70.000 en una tienda. Se ofrece una oferta, al comprar dos o más pares de zapatos del mismo precio se descuenta un 10% en cada par y por tres o más pantalones del mismo precio un 15% en cada pantalón. Juan paga por tres pantalones $ 38.250 y luego, compra dos pares de zapatos. ¿Cuánto pagó Juan por los dos pares de zapatos?

Answers

Answer:

49.5 $

Step-by-step explanation:

Comencemos por lo siguiente:

Si llamamos  x al precio del  par de zapatos  y  "y"  al precio de cada pantalón, entonces la tienda ofrece:

un par de zapatos   x    más dos pantalones   2*y  en  70.000 $  (deben ser 70 $ pero esa cifra no importa a los fines de resolver el problema, habrá solo que hacer el ajuste correspondiente, en lo que aqui respecta hablaremos de 70 $)

Entonces  de acuerdo a lo expuesto

x  +  2y  =  70         or    y  =  ( 70 - x) /2      (1)

Esa es la condición inicial.

Ahora bién  Juan paga por tres pantalones  38.25 $

Eso quiere decir ( como los pantalones son iguales) que pago 38.25/3 por cada pantalón

38.25/3  =  12.75 $

Ahora bién ese precio tubo una rebaja del precio original del 15 % ( quiere decir que el precio original de cada pantalón es de

12.75/0.85  =  15 $            x =  15 $

Entonces si nos vamos a la ecuación (1) (alli los precios son originales) tenemos que

y  =  ( 70  - x ) / 2       y  =  ( 70 - 15 ) / 2    y  = 55/2     y  =  27.5 $

Ahora Juan paga por dos pares de zapatos donde según las ofertas de la tienda el debera tener un descuento del 10 % en cada par

Por lo que si  un par cuesta  27.5 $  le darán un descuento del 10% es decir pagará  27.5 *0.9 =   24.75  por cada par, como son dos pares

pagará 2 * 24.75  =   49.5 $

Find the other endpoint of the line segment with the given endpoint and midpoint. I
nearest tenth.
Endpoint (-8, 8) Midpoint (5,-3)

Answers

The answer is (18,-14)

To get from (-8,8) to (5,-3) we can make into a transformation. T(x,y) = (x + 13,y - 11), so we just apply that transformation a second time to get (18, -14).

I do not understand any of this

Answers

Answer:

to get the slope. take the change of y axis value divide by the change of x axis value

Talia wants to find the amount of sugar that will fit in a box that is 4 inches long, 2 inches wide, and 6 inches high. What measurement does she need to calculate?

A)the perimeter of the box
B)the volume of the box
C)the width of the box
D)the area of the box

Answers

Answer:

B)the volume of the box

Explanation:

The volume represents the measurement of the inside of the box which is where the sugar will go.

Answer:

C) the volume of the box

:)

Twice a number minus 25 is less than 89. Translate it into an inequality and find the solution

Answers

Answer:

2x-25<89

x<57

Step-by-step explanation:

2x-25<89

2x<89+25

2x<114

Divide by 2...

x<57

Hope this helped! Please mark brainliest :)

will make brainliest, please help​

Answers

-5x+4y=2
9x-4y=6

eliminate the 4y as a positive 4 + negative 4 =0
this will leave you with
-5x=2
9x=6
add them
4x=8
divide both sides by 4

x=2


to find y, plug in x=2 for any one of the original equations

-5(2)+4y=2
-10+4y=2

add 10 to both sides

4y=12

divide both sides by 4

y=3

the final answer is (2,3)

What is the approximate value of x. Enter only a numeric answer rounded to the nearest tenth

Answers

Answer:

14.9 degrees

Step-by-step explanation:

For this problem you can use Tan∅=o/a and solve for the angle:

15.7 / 59 = 0.2661

Then use inverse tangent of 0.2661 to get 14.9.

Someone pls help me ill give out brainliest pls don’t answer if you don’t know

Answers

Answer:

Step-by-step explanation:

The area of a sector has the formula

[tex]A_s=\frac{\theta}{360}*\pi r^2[/tex] Hopefully, this looks somewhat familiar to you. Theta is the central angle given as 75, r is the radius given as 4. Filling in:

[tex]A_s=\frac{75}{360}*(3.14)(4)^2[/tex] and simplifying that a bit to look less threatening, but not by much:

[tex]A_s=\frac{5}{24}(3.14)(16)[/tex] and

[tex]A_s=\frac{251.2}{24}=\frac{157}{15}=10.466666666...[/tex] Not sure how you're supposed to express your answer so I gave both the fraction and its decimal equivalency.

CO
Evaluate the function for an input of 0.
x
f(x)
-2
0
IX-2
0
-1
3
3
O
4
4
1
3
2.
0

Answers

Answer:    4

Explanation:

Refer to row three of the table. The table shows that when x = 0, we have f(x) = 4. This is the same as saying f(0) = 4.

Answer:

the answer

-2 0-2

(17)^ 3 *(289)^ -6 =(17)^ 2m - 1​

Answers

Answer:

m = -4

Step-by-step explanation:

The given expression is :

[tex](17)^{3}\cdot(289)^{-6}=(17)^{2m-1}[/tex]

We need to find the value of m

We know that, 17² = 289

So,

[tex](17)^{3}\cdot(17^2)^{-6}=(17)^{2m-1}\\\\(17)^{3}\cdot(17)^{-12}=(17)^{2m-1}[/tex]

Also,[tex]x^ax^b=x^{a+b}[/tex]

So,

[tex]17^{3-12}=17^{2m-1}\\\\17^{-9}=17^{2m-1}\\\\\implies -9=2m-1\\\\-9+1=2m\\\\-8=2m\\\\m=-4[/tex]

So, the value of m is equal to -4.

A baseball diamond is a square that is 90 feet on each side. How far is it from home plate to second base? Round to the nearest hundredth.
90.50 feet
97.50 feet
107.28 feet
127.28 feet

Answers

Answer:

[tex]\sqrt{90^2 + 90^2}[/tex]

= 127.2792206

Step-by-step explanation:

Answer: 127.28 feet.

Step-by-step explanation:

It’s actually 127.2792206 feet, but rounded to the nearest hundredth, it’s 127.28 feet. Hope I helped!

Which of the following is a solution to 2sin^(2)x+sinx-1=0
45\deg
200\deg
240\deg
270\deg
330\deg

Answers

Hello,

[tex]2*sin^2(x)+sin(x)-1=0\\\Delta=1+4*2*1=9=3^2\\sin^2(x)=\frac{-1-3}{4} =-1\ (impossible)\\or\\sin^2(x)=\frac{-1+3}{4} =\dfrac{1}{2}\\sin(x)=\dfrac{\sqrt{2} }{2} \Longrightarrow (x=45^o\ or\ x=135^o)\\or\\sin(x)=-\dfrac{\sqrt{2} }{2} \Longrightarrow (x=315^o\ or\ x=225^o)\\[/tex]

Answer A : 45°

Which equation in slope-intercept form represents a line that passes through the point (5,−1) and is parallel to the line y=2x-7?

Answers

Answer:

y = 2x - 11

Step-by-step explanation:

y = 2x + b

-1 = 2(5) + b

-1 = 10 + b

-11 = b

bobby drove 110 miles and his car used 5 gallons of gas. How many miles can he drive with 16 gallos of gas

Answers

Answer:

Bobby can drive 352 miles with 16 gallons of gas.

Step-by-step explanation:

110 miles uses 5 gallons of gas.To find how many miles can be driven with one gallon we divide 110 by 5.

110 ÷ 5 = 22

Therefore to find the amount of miles that can be driven with 16 gallons we multiply 16 × 22 = 352

Help and explain pls and thankyouuu

Answers

Answer: (1, 8)

Step-by-step explanation:

Substitute y = 3x + 5 into the equation 2x + 3y = 26:

2x + 3(3x + 5) = 26

2x + 9x + 15 = 26

11x = 26 - 15

11x = 11

x = 1

Substitute the value of x into the equation y = 3x + 5:

y = 3(1) + 5 = 3 + 5 = 8

A television screen measures 35 cm wide and 26 cm high. What is the diagonal measure of the screen?

Answers

Answer:

43.6 (approximately)

Step-by-step explanation:

Diagonal measures of the screen is,

√(35²+26²)

= √(1901)

= 43.6 (approximately)

Answered by GAUTHMATH

We calculate the dioglonal using the Pythagorean theorem

[tex]\displaystyle\ C^2=A^2+B^2=> C=\sqrt{A^2+B^2} \ then \\\\C=\sqrt{26^2+35^2} =\sqrt{1225+676} =\sqrt{1901} \approx43.6\\\\Answer : the \:diagonal \:\:length is \:\underline{43.6}[/tex]

b) In a survey of 200 families, 80 were found using 'A brand ton only, 75 were found using 'B' brand tea only and each family is using at least one of the two brands, (1) Draw a Venn diagram to illustrate the above information (i) How many families are using both brande? (111) How many families are using 'A' brand? (iv) How many families are using 'B' brand?​

Answers

Answer:

The answer would be D

Step-by-step explanation:

I did the quiz

Step-by-step explanation:

n(u)=200

no(A)=80

no(B)=75

ii) Using both brand=200-80-75

=45

iii)n(A)=80+45

=125

iv)n(B)=75+45

=120

4x+9/y+11=0 and 6/y -3x=8​

Answers

9514 1404 393

Answer:

  (x, y) = (-2 12/17, -51)

Step-by-step explanation:

Here is the answer to ...

  [tex]4x+\dfrac{9}{y}+11=0\\\\\dfrac{6}{y}-3x=8[/tex]

If you mean something else, then parentheses are needed.

__

Let z = 1/y. Then the equations in general form are ...

  [tex]4x+9z+11=0\\3x-6z+8=0[/tex]

The solution (cross multiplication method) is ...

  x = (9·8 -(-6)·11)/(4(-6)-3·9) = 138/-51 = -2 12/17

  z = (11·3 -8·4)/-51 = -1/51

  y = 1/z = -51

The solution is (x, y) = (-2 12/17, -51).

determine which two values the following expression is between.

Answers

Answer:

Between 12.8 and 12.9

Step-by-step explanation:

Firstly, we would need to get the value of the surd

So we have this as;

2 √41 = 12.81

Now, looking at the options, we can see that it lies between 12.8 and 12.9; this is our answer

PLS HELP(algebra 1)
solve -7 + 18(17h + 19)

Answers

Answer:

335+306h

Step-by-step explanation:

solve the brackets first by distributing 18 to 17h and 19

-7+306h+342

then group the like terms

-7+342+306h

335+306h

Darcy's harvests 8 3/4 acres of corn every 5/6 hours. How much corn can she collect in a hour. ? =Amount of acres of corn done in a hour.

Answers

Answer:

Step-by-step explanation:

Acres of corn harvested : time taken = 8 3/4 : 5/6

How much corn can she collect in a hour. ?

Let

x = Acres of corn harvested

Acres of corn harvested : time taken = x : 1

8 3/4 : 5/6 = x : 1

35/4 : 5/6 = x : 1

35/4 ÷ 5/6 = x/1

35/4 × 6/5 = x/1

35*6/4*5 = x/1

210/20 = x/1

210 * 1 = 20 * x

210 = 20x

x = 210/20

= 10 1/2 acres

x = Acres of corn harvested = 10 1/2 acres

picture for question

Answers

Answer:

pretty sure it would be 90 degrees

In a triangle ABC, if a=16m c=14m b=19m. Find the angles​

Answers

Step-by-step explanation:

went to a calculator

it said

Angle ∠A = 55.543°

Angle ∠B = 78.28°

Angle ∠C = 46.177

what is a square plus b square?????????????????

Answers

Answer:

c squared

Step-by-step explanation:

Answer:

C Squared

Step-by-step explanation:

What is the volume of the sphere in terms of pi?
V=

6 in.

Answers

Answer:

= 288π in2

Step-by-step explanation:

Volume of a sphere is given by;

V =(4/3)πr3

V = volume

r is radius

Therefore V = {(4/3) × π × (6^3)} in3

= 288π in2

A savings account for a car is set up with an initial balance of $1500, and 250 is added every month (no other
transactions occur on the account). Write the expression: Write the equation equal to the $5,000 car goal: M=
1. How long will it take:

Answers

Answer:

ok so first we subtract 1500

5000-1500=3500

3500 divided by 250= 14

So it will take 14 months

Hope This Helps!!!

Other Questions
PLEASE SOMEONE HELP I need help ASAP!!! what is the circumference of a circle whose radius squared is 113 why your 21st century sophisticated, how do you explain the role of the plant in the change of the air in the jar I need help with this Luego de su cumpleaos, Benjamn ha decidido donar la tercera parte del dinero que recibi de regalo de sus familiares a una fundacin. Considerando las variables cantidad de dinero recibido por Benjamn y cantidad de dinero que donar Benjamn. a. Cul es la variable dependiente en esta situacin. (a) Define a goal for software product quality and an associated metric for that attribute. (b) Explain how you could show that the goal is validatable. (c) Explain how you could show that the goal is verifiable. 2. (a) Create a list of software product attributes and a list for software project attributes. (b) Rank these in the order of how difficult you consider them to be measured. (c) Discuss the reasons for your choices The Fun Committee is hosting the Annual City Festival. Jennifer is in charge of the committee and is planning a race to raise money for the Festival. The runners will earn money from donors for the number of miles they run. If the runners start at the park, run to City hall, and then run back to the park, how many total miles will each runner run? Show your work and leave your answer in simplest radical form if necessary. write 36,438 correct to 2 significant figure(show how you did it) Annual earnings, including bonuses, for Financial Analysts and Personal Financial Advisors, are currently following a skewed to the right distribution with a mean of $66,500 and a standard deviation of $10,500. According to the 68-95-99.7 rule, it is correct to say that (select ALL that apply):______. a. the middle 95% of all Financial Analysts and Personal Financial Advisors make between $45.500 and $77,000 annually. b. only 2.5% of all Financial Analysts and Personal Financial Advisors make less than $45,500 annually. c. both of the above statements are false Dada la funcin f(x)=1+6Sen(2x+/3) . Halle: Perodo, amplitud y desfase (1.5 puntos) Dominio y rango de la funcin (1.5 puntos) Grafique la funcin trigonomtrica (2 puntos) WILL GIVE BRAINLIEST PLZ!!! If producers expect the price of a good to decrease in the future, what will happen to the current equilibrium price and quantity of that good? a. The equilibrium price and quantity will both decrease. b. The equilibrium price and quantity will both increase. c. The equilibrium price will increase and equilibrium quantity will decrease. d. The equilibrium price will decrease and equilibrium quantity will increase. Which are important to consider when identifying the purpose of a speech? Select 4 C Corporation is investigating automating a process by purchasing a machine for $803,700 that would have a 9 year useful life and no salvage value. By automating the process, the company would save $138,500 per year in cash operating costs. The new machine would replace some old equipment that would be sold for scrap now, yielding $22,300. The annual depreciation on the new machine would be $89,300. The simple rate of return on the investment is closest to (Ignore income taxes.): Design an experiment to demonstrate phototropism. F(x) = x/2*8 what is f(x), when x=10 What happens in a flame test?A. Electrons are excited to higher energy levels and their emissionsare observed.B. Impurities in a sample are burned off in the flame until a puresample remainsC. Electrons are ejected from atoms when a sample is burned in theflame.D. A sample is melted in a flame to determine how much energy was Whic type of graph would you use if you wanted to see if one variable had an affect on another variable? Fix this grammatically incorrect sentence:The boats sails are very colorful. (There is ONLY ONE answer)A. The boats sails are very colorful.B. The boats sails are very colorful.C. The boats sails are very colorful.D. The boat sails are very colorful.E. The sails of the boat are colorful. comparacin entre el mito de la medusa y el mito cristiano de la creacin (argumenta) A particle is projected with a velocity of [tex]29.4ms^-^1[/tex] . Find it's maximum range on a horizontal plane through the point of projection.A.88.2m B.44.1m C.32.6m D.29.4m E.14.7m